Difference between revisions of "1993 AJHSME Problems/Problem 24"

(Problem)
m (Solution 2)
 
(13 intermediate revisions by 4 users not shown)
Line 3: Line 3:
 
What number is directly above <math>142</math> in this array of numbers?
 
What number is directly above <math>142</math> in this array of numbers?
  
<cmath> \begin{tabular}{cccccc}& & & 1 & &\\ & & 2 & 3 & 4 &\\ & 5 & 6 & 7 & 8 & 9\\ 10 & 11 & 12 &\cdots & &\\ \end{tabular} </cmath>
+
<cmath> \begin{array}{cccccc}& & & 1 & &\\ & & 2 & 3 & 4 &\\ & 5 & 6 & 7 & 8 & 9\\ 10 & 11 & 12 &\cdots & &\\ \end{array} </cmath>
  
 
<math>\text{(A)}\ 99 \qquad \text{(B)}\ 119 \qquad \text{(C)}\ 120 \qquad \text{(D)}\ 121 \qquad \text{(E)}\ 122</math>
 
<math>\text{(A)}\ 99 \qquad \text{(B)}\ 119 \qquad \text{(C)}\ 120 \qquad \text{(D)}\ 121 \qquad \text{(E)}\ 122</math>
 +
 +
==Solution==
 +
===Solution 1===
 +
Notice that a number in row <math>k</math> is <math>2k</math> less than the number directly below it. For example, <math>5</math>, which is in row <math>3</math>, is <math>(2)(3)=6</math> less than the number below it, <math>11</math>.
 +
 +
From row 1 to row <math>k</math>, there are <math>k \left(\frac{1+(-1+2k)}{2} \right) = k^2</math> numbers in those <math>k</math> rows. Because there are <math>12^2=144</math> numbers up to the 12th row, <math>142</math> is in the <math>k^{th}</math> row. The number directly above is in the 11th row, and is <math>22</math> less than <math>142</math>. Thus the number directly above <math>142</math> is <math>142-22=\boxed{\text{(C)}\ 120}</math>.
 +
 +
===Solution 2===
 +
Writing a couple more rows, the last number in each row ends in a perfect square. Thus <math>142</math> is two left from the last number in its row, <math>144</math>. One left and one up from <math>144</math> is the last number of its row, also a perfect square, and is <math>121</math>. This is one right and one up from <math>142</math>, so the number directly above <math>142</math> is one less than <math>121</math>, or <math>\boxed{\text{(C)}\ 120}</math>.
 +
 +
Remark: The last number in each row ends in a perfect square because there is an odd amount of numbers in each row and the sum of the first n odd numbers is always a perfect square.
 +
 +
===Solution 3 (eyeball it)===
 +
We can notice that even numbers are above even numbers so we can narrow our answers down to 2 solutions, <math>120</math> and <math>122</math>. Notice each number on the right end of each row is a perfect square. The perfect square closest to these is <math>121</math> so <math>122</math> is the first number on the next row. Notice <math>142</math> is the third to last number on its row so it is too far away from 122, so our answer is <math>\boxed{\text{(C)}\ 120}</math>.
 +
 +
==See Also==
 +
{{AJHSME box|year=1993|num-b=23|num-a=25}}
 +
{{MAA Notice}}

Latest revision as of 20:48, 17 January 2024

Problem

What number is directly above $142$ in this array of numbers?

\[\begin{array}{cccccc}& & & 1 & &\\ & & 2 & 3 & 4 &\\ & 5 & 6 & 7 & 8 & 9\\ 10 & 11 & 12 &\cdots & &\\ \end{array}\]

$\text{(A)}\ 99 \qquad \text{(B)}\ 119 \qquad \text{(C)}\ 120 \qquad \text{(D)}\ 121 \qquad \text{(E)}\ 122$

Solution

Solution 1

Notice that a number in row $k$ is $2k$ less than the number directly below it. For example, $5$, which is in row $3$, is $(2)(3)=6$ less than the number below it, $11$.

From row 1 to row $k$, there are $k \left(\frac{1+(-1+2k)}{2} \right) = k^2$ numbers in those $k$ rows. Because there are $12^2=144$ numbers up to the 12th row, $142$ is in the $k^{th}$ row. The number directly above is in the 11th row, and is $22$ less than $142$. Thus the number directly above $142$ is $142-22=\boxed{\text{(C)}\ 120}$.

Solution 2

Writing a couple more rows, the last number in each row ends in a perfect square. Thus $142$ is two left from the last number in its row, $144$. One left and one up from $144$ is the last number of its row, also a perfect square, and is $121$. This is one right and one up from $142$, so the number directly above $142$ is one less than $121$, or $\boxed{\text{(C)}\ 120}$.

Remark: The last number in each row ends in a perfect square because there is an odd amount of numbers in each row and the sum of the first n odd numbers is always a perfect square.

Solution 3 (eyeball it)

We can notice that even numbers are above even numbers so we can narrow our answers down to 2 solutions, $120$ and $122$. Notice each number on the right end of each row is a perfect square. The perfect square closest to these is $121$ so $122$ is the first number on the next row. Notice $142$ is the third to last number on its row so it is too far away from 122, so our answer is $\boxed{\text{(C)}\ 120}$.

See Also

1993 AJHSME (ProblemsAnswer KeyResources)
Preceded by
Problem 23
Followed by
Problem 25
1 2 3 4 5 6 7 8 9 10 11 12 13 14 15 16 17 18 19 20 21 22 23 24 25
All AJHSME/AMC 8 Problems and Solutions

The problems on this page are copyrighted by the Mathematical Association of America's American Mathematics Competitions. AMC logo.png